1
$\begingroup$

We were given an this question in my class:

Prove that a forest with n vertices and m components has n-m edges using induction on m.

Induction is not my strongest point and I was wondering if anyone could help me out with this?

  • 0
    The same question using a proof by contradiction was asked [here](http://math.stackexchange.com/questions/201611/graph-theory-forests).2013-04-12

2 Answers 2